Understanding the LaPlace Transformation for Solving Complex Integrals

Click For Summary
The discussion centers on a member's difficulty with the Laplace transformation, specifically in solving the integral of sin(t) multiplied by e^(-st). The member has attempted integration by parts but has not reached the expected answer. Other participants clarify that they cannot provide direct solutions but can offer hints, confirming that the member's final answer is indeed correct. An alternative method suggested involves expressing sin(t) in terms of exponential functions for easier integration. The conversation emphasizes the importance of understanding the transformation process rather than just obtaining the answer.
williamcarter
Messages
153
Reaction score
4
Moved from a technical forum, so homework template missing
Dear PF members.
I am requesting again your help as I keep struggling with the LaPlace transformation.
I have this exercise to do(please see below)
Capture.PNG


We know that L[f(t)]= integral from 0 to infinity of f(t)*e^(-st) dt
thus in our case, L[f(t)]= integral from 0 to infinity of sin(t)*e^(-st) dt

I tried doing it by parts twice, however I can't reach their answer.Please look below
Capture2.PNG

Could you please show me how to solve it?

Thank you in advance.
 
Physics news on Phys.org
Which part are you struggling with? Isn't your second attachment already the worked-out solution?
 
  • Like
Likes williamcarter
williamcarter said:
Dear PF members.
I am requesting again your help as I keep struggling with the LaPlace transformation.
I have this exercise to do(please see below)
View attachment 109363

We know that L[f(t)]= integral from 0 to infinity of f(t)*e^(-st) dt
thus in our case, L[f(t)]= integral from 0 to infinity of sin(t)*e^(-st) dt

I tried doing it by parts twice, however I can't reach their answer.Please look below
View attachment 109364
Could you please show me how to solve it?

Thank you in advance.

It is against PF rules for us to "show you how to solve it". At most, we can offer hints. However, since you have already done all the work, I can tell you that your final answer is correct.

Furthermore, the way you did it is one of the ways I would have done it; the other way would have been to write ##\sin(t) = (e^{it} - e^{-it})/(2i)## and integrate the two terms separately.
 
  • Like
Likes williamcarter
Thank you very much for confirming.
 
Question: A clock's minute hand has length 4 and its hour hand has length 3. What is the distance between the tips at the moment when it is increasing most rapidly?(Putnam Exam Question) Answer: Making assumption that both the hands moves at constant angular velocities, the answer is ## \sqrt{7} .## But don't you think this assumption is somewhat doubtful and wrong?

Similar threads

Replies
2
Views
2K
  • · Replies 1 ·
Replies
1
Views
1K
  • · Replies 3 ·
Replies
3
Views
3K
  • · Replies 10 ·
Replies
10
Views
2K
  • · Replies 3 ·
Replies
3
Views
1K
  • · Replies 4 ·
Replies
4
Views
3K
  • · Replies 1 ·
Replies
1
Views
1K
  • · Replies 2 ·
Replies
2
Views
2K
  • · Replies 4 ·
Replies
4
Views
3K
Replies
3
Views
2K